7.2 复数的四则运算

您所在的位置:网站首页 两个复数相减的绝对值几何意义 7.2 复数的四则运算

7.2 复数的四则运算

2024-07-16 15:53| 来源: 网络整理| 查看: 265

\({\color{Red}{欢迎到学科网下载资料学习 }}\) [ 【基础过关系列】高一数学同步精品讲义与分层练习(人教A版2019)] (https://www.zxxk.com/docpack/2921718.html) \({\color{Red}{ 跟贵哥学数学,so \quad easy!}}\)

必修第二册同步巩固,难度2颗星!

基础知识 复数的加、减法及其几何意义

(1) 复数的加法 设\(z_1=a+bi\),\(z_2=c+di\) ,\(a\) ,\(b\), \(c\),\(d∈R\) \(z_1+ z_2=a+b i+c+d i=(a+c)+(b+d) i\) 解释 ① 两个复数的和仍然是一个确定复数,形式类似两个多项式相加. ② 复数的加法满足交换律和结合律, 即对任意\(z_1\),\(z_2\),\(z_3∈C\),有\(z_1+z_2=z_2+z_1\),\((z_1+z_2 )+z_3=z_1+(z_2+z_3 )\). 【例】若\(z_1=-1+2i\),\(z_2=2-4i\),则\(z_1+ z_2=\) \(\underline{\quad \quad}\) . 解 \(z_1+ z_2=-1+2i+2-4i=1-2i\).  

(2) 复数加法的几何意义 复数与复平面内以原点为起点的向量一一对应的,设\(\overrightarrow{O Z_1}\),\(\overrightarrow{O Z_2}\)分别与复数\(z_1=a+bi\),\(z_2=c+di\)对应,则\(\overrightarrow{O Z_1} =(a,b)\),\(\overrightarrow{O Z_2} =(c,d)\),进而可得\(\overrightarrow{O Z_1} +\overrightarrow{O Z_2} =(a+c,b+d)\), 这说明两个向量\(\overrightarrow{O Z_1}\) 和\(\overrightarrow{O Z_2}\)的和就是与复数\((a+c)+(b+d) i\)对应的向量. 因此,复数的加法可以按照向量的加法来进行(符合平行四边形法则、向量坐标的运算),这就是复数加法的几何意义. image.png  

(3) 复数的减法 设\(z_1=a+bi\),\(z_2=c+di\) , \(a\) ,\(b\), \(c\),\(d∈R\), \(z_1- z_2=a+b i-(c+d i)=(a-c)+(b-d) i\). 解释 与实数减法的意义进行类比可得,减法的几何意义也是可以按照向量的减法来进行.  

复数的乘、除运算

(1)设\(z_1=a+bi\),\(z_2=c+di\) , \(a\) ,\(b\), \(c\),\(d∈R\), ① \(z_1⋅ z_2=(a+b i)⋅(c+d i)=(a c-b d)+(b c+a d) i\); ② \(\dfrac{z_1}{z_2}=\dfrac{a+b i}{c+d i}=\dfrac{(a+b i)(c-d i)}{(c+d i) \cdot(c-d i)}=\dfrac{(a c+b d)+(b c-a d) i}{c^2+d^2}\). 解释 ① 复数的乘法类似两个多项式相乘; ② 复数的乘法满足交换律、结合律,乘法对加法满足分配律, 即对任意\(z_1\),\(z_2\),\(z_3∈C\),有\(z_1 z_2=z_2 z_1\),\((z_1 z_2 ) z_3=z_1 (z_2 z_3 )\),\(z_1 (z_2+z_3 )=z_1 z_2+z_1 z_3\); ③ 复数的除法,分子分母都乘以分母的共轭复数\(c-d i\)再化简,其实就是“分母实数化”,好像初中学二次根式的“分母有理化”.  

【例】计算 \(\dfrac{2 i}{3+4 i}\). 解 \(\dfrac{2 i}{3+4 i}=\dfrac{2 i(3-4 i)}{(3+4 i)(3-4 i)}=\dfrac{6 i-8 i^2}{3^2-(4 i)^2}=\dfrac{8+6 i}{9+16}=\dfrac{8+6 i}{25}=\dfrac{8}{25}+\dfrac{6}{25} i\).  

基本方法 【题型1】 复数的四则运算

【典题1】计算下列各题   (1) \((2-i)(-1+5i)(3-4i)+2i\); \(\qquad \qquad\) (2) \(\dfrac{(1-4 i)(1+i)+2+4 i}{3+4 i}\); 解析(1)\((2-i)(-1+5i)+2i\) \(=-2+10i+i-5i^2+2i\) \(=-2+11i+5+2i\) \(=3+13i\); (2)\(\dfrac{(1-4 i)(1+i)+2+4 i}{3+4 i}\) \(=\dfrac{\left(1+i-4 i-4 i^2\right)+2+4 i}{3+4 i}=\dfrac{5-3 i+2+4 i}{3+4 i}\) \(=\dfrac{7+i}{3+4 i}=\dfrac{(7+i)(3-4 i)}{(3+4 i)(3-4 i)}=\dfrac{21-28 i+3 i-4 i^2}{25}\) \(=\dfrac{25-25 i}{25}=1-i\); 点拨 复数的四则运算类似多项式的四则运算,注意复数除法“分母实数化”,最终把结果化简为\(a+bi(a,b∈R)\)的形式.  

【典题2】设复数\(z\)满足 \(\dfrac{z+1}{z}=i\),则下列说法正确的是(  )  A.\(z\)为纯虚数 \(\qquad \qquad \qquad \qquad \qquad \qquad \qquad\) B.\(z\)的虚部为\(-\dfrac{1}{2}i\)  C.在复平面内,\(z\)对应的点位于第二象限 \(\qquad \qquad\) D.\(|z|=\dfrac{\sqrt{2}}{2}\) 解析\(\because z+1=zi\), \(\therefore(1-\mathrm{i}) \mathrm{z}=-1 \Longrightarrow \mathrm{z}=\dfrac{1}{-1+i}=-\dfrac{1}{2}-\dfrac{1}{2} i\). \(\therefore |z|=\dfrac{\sqrt{2}}{2}\),复数\(z\)的虚部为\(-\dfrac{1}{2}\),不是纯虚数, 复数\(z\)在复平面内所对应的点的坐标为\(\left(-\dfrac{1}{2},-\dfrac{1}{2}\right)\),在第三象限. \(\therefore\)正确的是\(D\). 故选 \(D\). 点拨 先把复数\(z\)求出或化简为\(a+bi(a,b∈R)\)的形式,在作出判断.  

【典题3】 若 \(f(z)=2 z+\bar{z}-3 i\), \(f(\bar{z}+i)=6-3 i\),求复数\(z\). 解析 \(\because f(z)=2z+\bar{z}-3i\), \(\therefore f(\bar{z}+i)=2(\bar{z}+i)+(\bar{z}+i)-3i\)\(=2\bar{z}+2i+z-i-3i=2\bar{z}+z-2i\). 又\(f(\bar{z}+i)=6-3i\),\(\therefore 2\bar{z}+z-2i=6-3i\), 即\(2\bar{z}+z=6-i\). 设\(z=x+yi(x,y∈R)\),则\(\bar{z}=x-yi\). \(\therefore 2(x-yi)+x+yi=3x-yi=6-i\), \(\therefore\left\{\begin{array} { l } { 3 x = 6 } \\ { y = 1 } \end{array} \Rightarrow \left\{\begin{array}{l} x=2 \\ y=1 \end{array}\right.\right.\),\(\therefore z=2+i\). 点拨 求满足某些要求的复数\(z\),可用待定系数法.  

【典题4】 在复数范围内解方程\(x^2+x+1=0\). 解析\(\because x^2+x+1=0\),\(\therefore\)配方得 \(\left(x+\dfrac{1}{2}\right)^2=-\dfrac{3}{4}\), \(\therefore x+\dfrac{1}{2}= \pm \dfrac{\sqrt{3}}{2} i\),即 \(x_1=-\dfrac{1}{2}-\dfrac{\sqrt{3}}{2} i\), \(x_2=-\dfrac{1}{2}+\dfrac{\sqrt{3}}{2} i\). 点拨 在复数范围内,实系数一元二次方程\(ax^2+bx+c=0(a≠0)\)的求根公式为: (1)当 \(\Delta \geqslant 0\)时, \(x=\dfrac{-b \pm \sqrt{b^2-4 a c}}{2 a}\);(2)当\(\Delta0)\)表示以\((a ,b)\)为圆心,\(r\)为半径的圆.  

【巩固练习】

1.在平行四边形\(ABCD\)中,对角线\(AC\)与\(BD\)相交于点\(O\),若向量 \(\overrightarrow{O A}\), \(\overrightarrow{O B}\)对应的复数分别是\(3+i\),\(-1+3i\),则 \(\overrightarrow{CD}\)对应的复数是(  )  A.\(2+4i\) \(\qquad \qquad \qquad\) B.\(-2+4i\) \(\qquad \qquad \qquad\) C.\(-4+2i\) \(\qquad \qquad \qquad\) D.\(4-2i\)  

2.复数\(z_1=1+2i\),\(z_2=-2+i\),\(z_3=-1-2i\),它们在复平面上的对应点是一个正方形的三个顶点,求这个正方形的第四个顶点对应的复数.  

3.如图,向量\(\overrightarrow{O Z_1}\) ,\(\overrightarrow{O Z_2}\)对应复数分别为\(z_1=a+bi(a,b∈R)\),\(z_2=c+di(c,d∈R)\),作出\(z_1+z_2\)对应的向量\(\overrightarrow{O Z }\),并指出\(|z_1+z_2 |≤|z_1 |+|z_2 |\)成立吗? image.png  

4.若\(z∈C\)且\(|z+2-2i|=1\),则\(|z-2-2i|\)的最小值是\(\underline{\quad \quad}\).  

5.已知三个复数\(z_1\),\(z_2\),\(z_3\),并且\(|z_1 |=|z_2 |=|z_3 |=1\),\(z_1\),\(z_2\)所对应的向量\(\overrightarrow{O Z_1}\) ,\(\overrightarrow{O Z_2}\)满足 \(\overrightarrow{O Z}_1 \cdot \overrightarrow{O Z}_2=0\),求\(|z_1+z_2-z_3 |\)的取值范围.  

6.已知\(A (1,2)\),\(B(a,1)\),\(C(2,3)\),\(D(-1,b)(a,b∈R)\)是复平面上的四个点,且向量 \(\overrightarrow{A B}\), \(\overrightarrow{C D}\)对应的复数分别为\(z_1\),\(z_2\).   (1)若\(z_1+z_2=1+i\),求\(z_1\),\(z_2\);   (2)若\(|z_1+z_2 |=2\),\(z_1-z_2\)为实数,求\(a\),\(b\)的值.      

参考答案

答案 \(D\) 解析 由于 \(\overrightarrow{C D}=\overrightarrow{B A}=\overrightarrow{O A}-\overrightarrow{O B}\),所以\(\overrightarrow{CD}\)对应的复数为\((3+i)-(-1+3i)=4-2i\).

答案 \(2-i\) 解析解法一:如图,设复数\(z_1\),\(z_2\),\(z_3\)所对应的点分别为\(A\),\(B\),\(C\),正方形的第四个顶点\(D\)对应的复数为\(x+yi(x,y∈R)\). image.png 则 \(\overrightarrow{A D}=\overrightarrow{O D}-\overrightarrow{O A}=(x+y i)-(1+2 i)=(x-1)+(y-2) i\), \(\overrightarrow{B C}=\overrightarrow{O C}-\overrightarrow{O B}=(-1-2 i)-(-2+i)=1-3 i\). 因为 \(\overrightarrow{A D}=\overrightarrow{B C}\), 所以\((x-1)+(y-2)i=1-3i\), 所以 \(\left\{\begin{array}{l} x-1=1 \\ y-2=-3 \end{array}\right.\),解得 \(\left\{\begin{array}{l} x=2 \\ y=-1 \end{array}\right.\). 故点\(D\)对应的复数为\(2-i\). 解法二:设复数\(z_1\),\(z_2\),\(z_3\)所对应的点分别为\(A\),\(B\),\(C\), 正方形的第四个顶点\(D\)对应的复数为\(x+yi(x,y∈R)\). 因为点\(A\)与点\(C\)关于原点对称, 所以原点\(O\)为正方形的中心, 所以点\(O\)也是点\(B\)与点\(D\)连线的中点, 于是\((-2+i)+(x+yi)=0\), 所以x=2,y=-1, 故点\(D\)对应的复数为\(2-i\).

答案 成立 解析 解法1:由向量平行四边形法则知,分别以向量\(\overrightarrow{O Z_1}\) ,\(\overrightarrow{O Z_2}\)为邻边作平行 四边形所得的对角线\(OZ\),即为向量\(\overrightarrow{O Z }\),如图(1). image.png 解法2:以向量\(\overrightarrow{O Z_1}\)的终点\(Z_1\)为起点作向量 \(\overrightarrow{Z_1 Z}=\overrightarrow{O Z_2}\), 则向量\(\overrightarrow{O Z }\)即为复数\(z_1+z_2\)对应的向量,如图(2). 由向量模的性质知:\(|z_1+z_2 |≤|z_1 |+|z_2 |\)成立.

答案 \(3\) 解析 已知\(|z-(-2+2i)|=1\)中, \(z\)的对应点轨迹是以\((-2,2)\)为圆心,\(1\)为半径的圆, \(|z-(2+2i)|\)表示圆上的点与点\((2,2)\)之间的距离, 最小值为圆心与点\((2,2)\)的距离减去半径,易得\(|z-2-2i|\)的最小值为\(3\).

答案 \([\sqrt{2}-1,\sqrt{2}+1]\) 解析 由题意可知 复数\(z_1\),\(z_2\),\(z_3\)对应的点\(Z_1\),\(Z_2\),\(Z_3\)在单位圆上, 又\(\overrightarrow{O Z}_1 \cdot \overrightarrow{O Z}_2=0\),\(\therefore OZ_1⊥OZ_2\). 不妨设\(Z_1 (1,0)\),\(Z_2 (0,1)\),如图 image.png \(\therefore\)当\(Z_3\)与\(A\)重合时,\(|z_1+z_2-z_3 |\)有最小值为\(\sqrt{2}-1\); 当\(Z_3\) 与\(B\)重合时,\(|z_1+z_2-z_3 |\)有最大值为\(\sqrt{2}+1\). \(\therefore |z_1+z_2-z_3 |\)的取值范围是\([\sqrt{2}-1,\sqrt{2}+1]\). 故答案为\([\sqrt{2}-1,\sqrt{2}+1]\).

答案 (1) \(z_1=4-i\),\(z_2=-3+2i\);(2) \(a=4\),\(b=2\). 解析(1)向量 \(\overrightarrow{A B}=(a-1,-1)\), \(\overrightarrow{C D}=(-3, b-3)\)对应的复数分别为 \(z_1=(a-1)-i\),\(z_2=-3+(b-3)i\). \(\therefore z_1+z_2=(a-4)+(b-4)i=1+i\). \(\therefore a-4=1\),\(b-4=1\).解得\(a=b=5\). \(\therefore z_1=4-i\),\(z_2=-3+2i\). (2)\(|z_1+z_2 |=2\),\(z_1-z_2\)为实数, \(\therefore \sqrt{(a-4)^2+(b-4)^2}=2\),\((a+2)+(2-b)i∈R\), \(\therefore 2-b=0\),解得\(b=2\), \(\therefore (a-4)^2+4=4\),解得\(a=4\). \(\therefore a=4\),\(b=2\).  

分层练习 【A组---基础题】

1.若\(z_1=2+i\),\(z_2=3+ai(a∈R)\),\(z_1+z_2\)所对应的点在实轴上,则\(a\)为(  )  A.\(3\) \(\qquad \qquad \qquad \qquad\) B.\(2\) \(\qquad \qquad \qquad \qquad\) C.\(1\) \(\qquad \qquad \qquad \qquad\) D.\(-1\)  

2.在复平面内,复数\(1+i\)与\(1+3i\)分别对应向量 \(\overrightarrow{O A}\)和 \(\overrightarrow{O B}\),其中\(O\)为坐标原点,则 \(|\overrightarrow{A B}|=\) (  )  A.\(\sqrt{2}\) \(\qquad \qquad \qquad \qquad\) B.\(2\) \(\qquad \qquad \qquad \qquad\) C. \(\sqrt{10}\) \(\qquad \qquad \qquad \qquad\) D.\(4\)  

3.若\(|z-1|=|z+1|\),则复数\(z\)对应的点\(Z\)(  )  A.在实轴上 \(\qquad \qquad \qquad\) B.在虚轴上 \(\qquad \qquad \qquad\) C.在第一象限 \(\qquad \qquad \qquad\) D.在第二象限  

4.设\(z_1\),\(z_2\)是复数,给出四个命题 ①若\(|z_1-z_2 |=0\),则 \(\overline{z_1}=\overline{z_2}\) \(\qquad \qquad\) ②若 \(z_1=\overline{z_2}\) ,则 \(\overline{z_1}=z_2\) ③若\(|z_1 |=|z_2 |\),则 \(z_1 \cdot \overline{z_1}=z_2 \cdot \overline{z_2}\) \(\qquad\) ④若\(|z_1 |=|z_2 |\),则 \(|z_1 |=|z_2 |\) 其中真命题的个数有(  )  A.\(1\) \(\qquad \qquad \qquad \qquad\) B.\(2\) \(\qquad \qquad \qquad \qquad\) C.\(3\) \(\qquad \qquad \qquad \qquad\) D.\(4\)  

5.已知复数\(z=\dfrac{8-i}{2+3 i}\)(\(i\)为虚数单位),下列说法 其中正确的有(  ) ①复数\(z\)在复平面内对应的点在第四象限;②\(|z|=\sqrt{5}\); ③\(z\)的虛部为\(-2i\); ④ \(\bar{z}=1-2i\).  A.\(1\)个 \(\qquad \qquad \qquad \qquad\) B.\(2\)个 \(\qquad \qquad \qquad \qquad\) C.\(3\)个\(\qquad \qquad \qquad \qquad\) D.\(4\)个  

6.若\(|z|=3\),\(z+\bar{z}=0\),则复数\(z=\)\(\underline{\quad \quad}\).  

7.设\(x\),\(y\)为实数,且 \(\dfrac{x}{1-i}+\dfrac{y}{1-2 i}=\dfrac{5}{1-3 i}\),则\(x+y=\) \(\underline{\quad \quad}\) .  

8.方程\(x^2+2x-1=0\)在复数范围内的解是 \(\underline{\quad \quad}\).  

9.已知复数\(z_1=\left(a^2-2\right)+(a-4) i\),\(z_2=a-\left(a^2-2\right) i(a \in R)\),且\(z_1-z_2\)为纯虚数,则\(a=\)\(\underline{\quad \quad}\) .  

10.已知\(|z|=2\),则\(|z-i|\)的最大值为\(\underline{\quad \quad}\).  

11.如果复数\(z\)满足\(|z+3i|+|z-3i|=6\),那么\(|z+1+i|\)的最小值是\(\underline{\quad \quad}\) .  

12.满足\(z+\dfrac{5}{z}\)是实数,且\(z+3\)的实部与虚部是相反数的虚数\(z\)是否存在?若存在,求出虚数z;若不存在,请说明理由.  

参考答案

答案 \(D\) 解析 \(z_1+z_2=(2+i)+(3+ai)=5+(a+1)i\),\(z_1+z_2\)对应的点在实轴上, 即\(z_1+z_2\)为实数,因此\(a+1=0\),\(a=-1\).

答案 \(B\) 解析\(\overrightarrow{A B}\)对应的复数为\((1+3i)-(1+i)=2i\),故 \(|\overrightarrow{A B}|=|2 i|=2\).

答案 \(B\) 解析 由\(|z-1|=|z+1|\)知\(z\)对应的点的轨迹是两点\((1,0)\),\((-1,0)\)连线的垂直平分线,即虚轴.

答案 \(C\) 解析由\(z_1\),\(z_2\)是复数,得 在①中,若\(|z_1-z_2 |=0\),则\(z_1\),\(z_2\)的实部和虚部都相等, \(\therefore \overline{z_1}=\overline{z_2}\),故①正确; 在②中,若 \(z_1=\overline{z_2}\),则\(z_1\),\(z_2\)的实数相等,虚部互为相反数, \(\therefore \overline{z_1}=z_2\),故②正确; 在③中,若\(|z_1 |=|z_2 |\),则 \(\left.\left|z_1 \cdot \overline{z_1}=z_2 \cdot \overline{z_2}=\right| z_1\right|^2\),故③正确; 在④中,若\(|z_1 |=|z_2 |\),则由复数的模的性质得 \(z_1^2 \neq z_2^2\), 如\(|1-i|=|1+i|=\sqrt{2}\),但\((1-i)^2=-2i≠(1+i)^2=2i\),故④不正确. 故选 \(C\).

答案 \(B\) 解析 \(\because z=\dfrac{8-i}{2+3 i}=\dfrac{(8-i)(2-3 i)}{(2+3 i)(2-3 i)}=\dfrac{13-26 i}{13}=1-2 i\), \(\therefore\)复数\(z\)在复平面内对应的点的坐标为\((1,-2)\),在第四象限; \(|z|=\sqrt{5}\);\(z\)的虚部为\(-2\);\(\bar{z}=1+2i\). 故①②正确;③④错误. 故选\(B\).

答案 \(3i\)或\(-3i\) 解析设\(z=x+yi(x,y∈R)\),则有\(\bar{z}=x-yi\), 因此 \(\left\{\begin{array}{l} x^2+y^2=9 \\ x+y i+x-y i=0 \end{array}\right.\),解得\(\left\{\begin{array}{l} x=0 \\ y=3 \end{array}\right.\)或 \(\left\{\begin{array}{l} x=0, \\ y=-3 \end{array}\right.\). \(\therefore z=3i\)或\(-3i\).

答案 \(4\) 解析 \(\dfrac{x}{1-i}+\dfrac{y}{1-2 i}=\dfrac{5}{1-3 i}\)\(\Rightarrow \dfrac{x(1+i)}{(1-i)(1+i)}+\dfrac{y(1+2 i)}{(1+2 i)(1-2 i)}=\dfrac{5(1+3 i)}{(1-3 i)(1+3 i)}\) \(\Rightarrow \dfrac{1}{2} x(1+i)+\dfrac{1}{5} y(1+2 i)=\dfrac{1}{2}(1+3 i) \Rightarrow\)\(\left\{\begin{array}{l} \dfrac{1}{2} x+\dfrac{1}{5} y=\dfrac{1}{2} \\ \dfrac{1}{2} x+\dfrac{2}{5} y=\dfrac{3}{2} \end{array}\right.\), 解得 \(\left\{\begin{array}{l} x=-1 \\ y=5 \end{array}\right.\), \(\therefore x+y=4\).

答案 \(-1+i\)或\(-1-i\) 解析 \(\because x^2+2x+2=0\),\(\therefore\) 配方得\((x+1)^2=-1\), \(\therefore x+1=±i\),即\(x_1=-1-i\),\(x_2=-1+i\).

答案 \(-1\) 解析 \(z_1-z_2=\left(a^2-a-2\right)+\left(a-4+a^2-2\right) i(a \in R)\)为纯虚数, \(\therefore\left\{\begin{array}{l} a^2-a-2=0 \\ a^2+a-6 \neq 0 \end{array}\right.\),解得\(a=-1\).

答案 \(3\) 解析依题意\(|z|=2\),所以\(z\)对应的点在以原点为圆心,\(2\)为半径的圆上, 而\(|z-i|\)表示z对应的点与\((0,1)\)点间的距离,显然这个距离的最大值是\(1+2=3\).

答案 \(1\) 解析复数\(z\)满足\(|z+3i|+|z-3i|=6\), \(\therefore z\)的几何意义是以\(A(0,3)\),\(B(0,-3)\)为端点的线段\(AB\), 则\(|z+1+i|=|z-(-1-i)|\)的几何意义为\(AB\)上的点到\(C(-1,-1)\)的距离, 则由图象知\(C\)到线段\(AB\)的距离的最小值为\(1\), 故答案为 \(1\).

答案 存在虚数\(z=-1-2i\)或\(z=-2-i\)满足题设条件 解析设虚数\(z=x+yi(x,y∈R\),且\(y≠0)\), 则 \(z+\dfrac{5}{z}=x+y i+\dfrac{5}{x+y i}=x+\dfrac{5 x}{x^2+y^2}+\left(y-\dfrac{5 y}{x^2+y^2}\right) i\), \(z+3=x+3+y i\), 由已知得 \(\left\{\begin{array}{l} y-\dfrac{5 y}{x^2+y^2}=0 \\ x+3=-y \end{array}\right.\),又\(\because y≠0\), \(\therefore\left\{\begin{array}{l} x^2+y^2=5 \\ x+y=-3 \end{array}\right.\),解得 \(\left\{\begin{array}{l} x=-1 \\ y=-2 \end{array}\right.\)或 \(\left\{\begin{array}{l} x=-2 \\ y=-1 \end{array}\right.\), \(\therefore\) 存在虚数\(z=-1-2i\)或\(z=-2-i\)满足题设条件.  

【B组---提高题】

1.已知 \(\dfrac{(1+i)^{2 n}}{1-i}+\dfrac{(1-i)^{2 n}}{1+i}=2^n\),则最小正整数\(n\)为\(\underline{\quad \quad}\) .  

2.已知\(|z_i |+|z_i-2|=3\),\(z_i∈C\),\(i=1,2\),\(|z_1-z_2 |=2\),则\(|z_1 |+|z_2 |\)的最大值为\(\underline{\quad \quad}\).  

3.设\(z\)是虚数, \(\omega=z+\dfrac{1}{z}\)是实数,且\(-1



【本文地址】


今日新闻


推荐新闻


CopyRight 2018-2019 办公设备维修网 版权所有 豫ICP备15022753号-3